What is Supremum: Definition and 144 Discussions

In mathematics, the infimum (abbreviated inf; plural infima) of a subset



S


{\displaystyle S}
of a partially ordered set



T


{\displaystyle T}
is the greatest element in



T


{\displaystyle T}
that is less than or equal to all elements of



S
,


{\displaystyle S,}
if such an element exists. Consequently, the term greatest lower bound (abbreviated as GLB) is also commonly used.The supremum (abbreviated sup; plural suprema) of a subset



S


{\displaystyle S}
of a partially ordered set



T


{\displaystyle T}
is the least element in



T


{\displaystyle T}
that is greater than or equal to all elements of



S
,


{\displaystyle S,}
if such an element exists. Consequently, the supremum is also referred to as the least upper bound (or LUB).The infimum is in a precise sense dual to the concept of a supremum. Infima and suprema of real numbers are common special cases that are important in analysis, and especially in Lebesgue integration. However, the general definitions remain valid in the more abstract setting of order theory where arbitrary partially ordered sets are considered.
The concepts of infimum and supremum are similar to minimum and maximum, but are more useful in analysis because they better characterize special sets which may have no minimum or maximum. For instance, the set of positive real numbers





R


+




{\displaystyle \mathbb {R} ^{+}}
(not including 0) does not have a minimum, because any given element of





R


+




{\displaystyle \mathbb {R} ^{+}}
could simply be divided in half resulting in a smaller number that is still in





R


+




{\displaystyle \mathbb {R} ^{+}}
. There is, however, exactly one infimum of the positive real numbers: 0, which is smaller than all the positive real numbers and greater than any other real number which could be used as a lower bound.

View More On Wikipedia.org
  1. L

    I Supremum of a set, relations and order

    Hello, found this proof online, I was wondering why they defined r_2=r_1-(r_1^2-2)/(r_1+2)? i understand the numerator, because if i did r_1^2-4 then there might be a chance that this becomes negative. But for the denominator, instead of plus 2, can i do plus 10 as well? or some other number...
  2. C

    Can You Solve a Problem Using the Definition of Supremum?

    For this problem, My solution: Using definition of Supremum, (a) ##M ≥ s## for all s (b) ## K ≥ s## for all s implying ##K ≥ M## ##M ≥ s## ##M + \epsilon ≥ s + \epsilon## ##K ≥ s + \epsilon## (Defintion of upper bound) ##K ≥ M ≥ s + \epsilon## (b) in definition of Supremum ##M ≥ s +...
  3. P

    I Showing equivalence of two definitions of essential supremum

    Assume ##f: X\to\mathbb R## to be a measurable function on a measure space ##(X,\mathcal A,\mu)##. The first definition is ##\operatorname*{ess\,sup}\limits_X f=\inf A##, where $$A=\{a\in\mathbb R: \mu\{x\in X:f(x)>a\}=0\}$$ and the second is ##\operatorname*{ess\,sup}\limits_X f=\inf B## where...
  4. chwala

    Find the supremum of ##Y## if it exists. Justify your answer.

    Refreshing on old university notes...phew, not sure on this... Ok in my take, ##x>0##, and ##\dfrac{dy}{dx} = -3x^2=0, ⇒x=0## therefore, ##(x,y)=(0,\sqrt2)## is a critical point. Further, ##\dfrac{d^2y}{dx^2}(x=0)=-6x=-6⋅0=0, ⇒f(x)## has an inflection at ##(x,y)=(0,\sqrt2)##. The supremum of...
  5. L

    Proving the Infimum and Supremum: A Short Guide for Scientists

    Hi, I have problems with the proof for task a I started with the supremum first, but the proof for the infimum would go the same way. I used an epsilon neighborhood for the proof I then argued as follows that for ##b- \epsilon## the following holds ##b- \epsilon < b## and ##b- \epsilon \in...
  6. S

    On the definition of radius of convergence; a small supremum technicality

    I am reading the following passage in these lecture notes (chapter 10, in the proof of theorem 10.3) on power series (and have seen similar statements in other texts): I'm confused about ##|x_0|<R##. If ##M=\sup (A)##, then for every ##M'<M##, there exists an ##x\in A## such that ##x>M'##...
  7. J

    I Show ##sup\{a \in \mathbb{Q}: a^2 \leq 3\} = \sqrt{3}##

    I would wish to receive verification for my proof that ##sup\{a \in \mathbb{Q}: a^2 \leq 3\} = \sqrt{3}##. • It is easy to verify that ##A = \{a \in \mathbb{Q}: a^2 \leq 3\} \neq \varnothing##. For instance, ##1 \in \mathbb{Q}, 1^2 \leq 3## whence ##1 \in A##. • We claim that ##\sqrt{3}## is an...
  8. M

    My proof of the Geometry-Real Analysis theorem

    Consider a convex shape ##S## of positive area ##A## inside the unit square. Let ##a≤1## be the supremum of all subsets of the unit square that can be obtained as disjoint union of finitely many scaled and translated copies of ##S##. Partition the square into ##n×n## smaller squares (see...
  9. Mathvsphysics

    A Limits and Supremum: Is It True?

    We have ##a_n## converges in norm to ##a## and a set ##S## such that for all ##n\ge 0## $$\sup_{s\in S} <a_n,s><+\infty .$$ Is it true that ##\sup_{s\in S} <a,s><+\infty##
  10. S

    MHB Finding the Infimum and Supremum

    Hello, I feel like I am struggling with this more than I should. I can tell intuitively what the infimum and supremum are, but I am pretty sure that I need a more formal proof style answer. How would one actually prove this question?
  11. yucheng

    Contradictory Proof of Supremum of E: Is it Circular?

    N.B. I have inserted the proof here as reference. See the bolded text. My question is, isn't the reasoning "##x^{2}+5 \varepsilon<2,## thus ##(x+\varepsilon)^{2}<2 .## " circular? If we can already find an ##0<\varepsilon<1## such that ##x^{2}+5 \varepsilon<2,## Can't one also claim that " we...
  12. C

    I Supremum proof & relation to Universal quantifier

    In the following proof: I didn't understand the following part: Isn't it supposed to be : ## a > s_A - \epsilon >0 ## and ## b > s_B - \epsilon >0 ## Because to prove that ## s ## is a supremum, we need to prove the following: For every ## \epsilon > 0 ## there exists ## m \in M ## such...
  13. N

    Find the infimum and/or supremum and see if the set is bounded

    ##S_3 = \left\{ \ x∈ℝ : x^2+x+1≥0 \right\}## I am not sure if I have done this correctly. The infimum/supremum and maximum/minimum are confusing me a bit. This is how I started: ##x^2+x+1=0## ##x^2+x+ \frac1 4\ =\frac{-3} {4}\ ## ## \left\{ x^2+\frac 1 2\ \right\} ^2 +\frac 3 4\ = 0##...
  14. AutGuy98

    MHB Proof of an Infimum Being Equal to the Negative Form of a Supremum ()

    Hey guys, I'm kind of in a rush because I'll have to go to my classes soon here at USF Tampa, but I had one last problem for Intermediate Analysis that needs assistance. Thank you in advance to anyone providing it. Question being asked: "Let $A$ be a nonempty set of real numbers which is...
  15. AutGuy98

    MHB Infimum and Supremum of a Set (Need Help Finding Them)

    Hey guys, I have this Intermediate Analysis problem that I need help finding the answer to. This is what the question asks: "Find the supremum and infimum of each of the following sets (considered as subsets of the real numbers). If a supremum or infimum doesn’t exist, then say so. No formal...
  16. Eclair_de_XII

    I Is there anything wrong with how the supremum of a set is written?

    I'm just having random thoughts today, and I didn't know where to put this, since this isn't even a homework problem. Anyway, is my way of writing the supremum of a set correct syntax-wise, or no?
  17. Math Amateur

    I Application of Supremum Property .... Garling, Remarks on Theorem 3.1.1

    I am reading D. J. H. Garling's book: "A Course in Mathematical Analysis: Volume I: Foundations and Elementary Real Analysis ... ... I am focused on Chapter 3: Convergent Sequences I need some help to fully understand some remarks by Garling made after the proof of Theorem 3.1.1...
  18. Math Amateur

    MHB Application of Supremum Property .... Garling, Theorem 3.1.1 ....

    I am reading D. J. H. Garling's book: "A Course in Mathematical Analysis: Volume I: Foundations and Elementary Real Analysis ... ... I am focused on Chapter 3: Convergent Sequences I need some help to fully understand the proof of Theorem 3.1.1 ...Garling's statement and proof of...
  19. V

    Finding an upper bound that is not the supremum

    I just want to see if I did this correctly, the interval (0,1) has 2 as an upper bound but the supremum of S is 1. So M would be equal to 2? Thank you.
  20. J

    MHB Real Analysis - Convergence to Essential Supremum

    Problem: Let $\left(X, M, \mu\right)$ be a probability space. Suppose $f \in L^\infty\left(\mu\right)$ and $\left| \left| f \right| \right|_\infty > 0$. Prove that $lim_{n \rightarrow \infty} \frac{\int_{X}^{}\left| f \right|^{n+1} \,d\mu}{\int_{X}^{}\left| f \right|^{n} \,d\mu} = \left| \left|...
  21. NihalRi

    What is the proof for the limit superior?

    Homework Statement 2. Relevant equation Below is the definition of the limit superior The Attempt at a Solution I tried to start by considering two cases, case 1 in which the sequence does not converge and case 2 in which the sequence converges and got stuck with the second case. I know...
  22. evinda

    MHB Finding Supremum and Infimum of Sets with Inequalities

    Hello! (Wave) I want to find the supremum, infimum of the following sets: $$\{ x \in \mathbb{R}: 0<x^2-1<3\}, \{1+\frac{(-1)^n}{n}: n=1,2, \dots \}$$ For the first set I have thought the following: $$ 0<x^2-1<3 \Rightarrow 1<x^2<4 \Rightarrow x^2>1 \text{ and } x^2 <4 \Rightarrow (x>1 \text{...
  23. ertagon2

    MHB Sequences and their limits, convergence, supremum etc.

    Could someone check if my answers are right and help me with question 5?
  24. E

    MHB Supremum and Infimum of Bounded Sets Multiplication

    Hey all, I started to learn this subject, and i understtod how to find the supremum and infimum of a given set or function. but I have problem with one question which I can not solve, and I don't know how to start. This is the quesion: Given to bounded sets X and Y, which their element are REAL...
  25. Math Amateur

    MHB Supremum Property (AoC) .... etc .... Yet a further question/Issue ....

    I am reading Houshang H. Sohrab's book: "Basic Real Analysis" (Second Edition). I am focused on Chapter 2: Sequences and Series of Real Numbers ... ... I need help with yet a further issue/problem with Theorem 2.1.45 concerning the Supremum Property (AoC), the Archimedean Property, and the...
  26. Math Amateur

    I Supremum Property (AoC) .... etc .... Yet a further question

    I am reading Houshang H. Sohrab's book: "Basic Real Analysis" (Second Edition). I am focused on Chapter 2: Sequences and Series of Real Numbers ... ... I need help with yet a further issue/problem with Theorem 2.1.45 concerning the Supremum Property (AoC), the Archimedean Property, and the...
  27. Math Amateur

    MHB Supremum Property (AoC) .... etc .... Another question/Issue ....

    I am reading Houshang H. Sohrab's book: "Basic Real Analysis" (Second Edition). I am focused on Chapter 2: Sequences and Series of Real Numbers ... ... I need help with another issue/problem with Theorem 2.1.45 concerning the Supremum Property (AoC), the Archimedean Property, and the Nested...
  28. Math Amateur

    I Supremum Property (AoC) .... etc .... Another question/Issue

    I am reading Houshang H. Sohrab's book: "Basic Real Analysis" (Second Edition). I am focused on Chapter 2: Sequences and Series of Real Numbers ... ... I need help with another issue/problem with Theorem 2.1.45 concerning the Supremum Property (AoC), the Archimedean Property, and the Nested...
  29. Math Amateur

    MHB Supremum Property (AoC), Archimedean Property, Nested Intervals Theorem ....

    I am reading Houshang H. Sohrab's book: "Basic Real Analysis" (Second Edition). I am focused on Chapter 2: Sequences and Series of Real Numbers ... ... I need help with Theorem 2.1.45 concerning the Supremum Property (AoC), the Archimedean Property, and the Nested Intervals Theorem ... ...
  30. Math Amateur

    I Supremum Property, Archimedean Property, Nested Intervals

    I am reading Houshang H. Sohrab's book: "Basic Real Analysis" (Second Edition). I am focused on Chapter 2: Sequences and Series of Real Numbers ... ... I need help with Theorem 2.1.45 concerning the Supremum Property (AoC), the Archimedean Property, and the Nested Intervals Theorem ... ...
  31. Bunny-chan

    Supremum and infimum of specific sets

    Homework Statement I'm in need of some help to be able to determine the supremum and infimum of the following sets:A = \left\{ {mn\over 1+ m+n} \mid m, n \in \mathbb N \right\}B = \left\{ {mn\over 4m^2+m+n^2} \mid m, n \in \mathbb N \right\}C = \left\{ {m\over \vert m\vert +n} \mid m \in...
  32. i_hate_math

    I Upper bound and supremum problem

    Claim: Let A be a non-empty subset of R+ = {x ∈ R : x > 0} which is bounded above, and let B = {x2 : x ∈ A}. Then sup(B) = sup(A)2. a. Prove the claim. b. Does the claim still hold if we replace R+ with R? Explain briefly. So I have spent the past hours trying to prove this claim using the...
  33. mr_persistance

    Apostol's Calculus I. 3.12 - Verify Solution.

    1. If x and y are arbitrary real numbers with x < y, prove that there is at least one real z satisfying x<z<y.2. I'll be using this theorem: T 1.32 Let h be a given positive number and let S be a set of real numbers. (a) If S has a supremum, then for some x in S we have x > sup S - h.The Attempt...
  34. RJLiberator

    Infimum and Supremum, when they Do not exist in finite sets

    Homework Statement Give an example of each, or state that the request is impossible: 1) A finite set that contains its infimum, but not its supremum. 2) A bounded subset of ℚ that contains its supremum, but not its infimum. Homework EquationsThe Attempt at a Solution I either understand this...
  35. I

    Rational numbers, supremum (Is my proof correct?)

    Homework Statement Show that the set ##\{x \in \mathbf Q; x^2< 2 \}## has no least upper bound in ##\mathbf Q##; using that if ##r## were one then ##r^2=2##. Do this assuming that the real field haven't been constructed. Homework Equations N/A The Attempt at a Solution Attempt at proof: ##r\in...
  36. I

    Supremum, Infimum (Is my proof correct?)

    Homework Statement Let ##A## be a nonempty set of real numbers which is bounded below. Let ##-A## be the set of all numbers ##-x##, where ##x \in A##. Prove that ##\inf A = -\sup(-A)##. Homework Equations Definition: Suppose ##S## is an ordered set, ##E\subset S##, and ##E## is bounded above...
  37. W

    I Supremum and Infimum Proof

    Let S and T be subsets of R such that s < t for each s ∈ S and each t ∈ T. Prove carefully that sup S ≤ inf T. Attempt: I start by using the definition for supremum and infinum, and let sup(S)= a and inf(T)= b i know that a> s and b< t for all s and t. How do i continue? , do i prove it...
  38. P

    I Supremum inside and outside a probability

    I'm trying to deal with the supremum concept in a specific situation, but I think I'm getting the concept wrong. A step of a proof I'm going through states: P\ [\sup\limits_{x}\ |f(x)\ -\ f'(x)|\ >\ y\ |\ z]\ \ \leq\ \ \sum_{i=1}^M\ P\ [\ |f(x)\ -\ f'(x)|\ >\ y\ |\ z]\ \ \leq\ \ M\times\...
  39. V

    Proof of points arbitrarily close to supremum

    Homework Statement Let S \subset \mathbb{R} be bounded above. Prove that s \in \mathbb{R} is the supremum of S iff. s is an upper bound of S and for all \epsilon > 0 , there exists x \in S such that |s - x| < \epsilon . Homework Equations **Assume I have only the basic proof...
  40. Alpharup

    B Doubt regarding least upper bound?

    I am using Spivak Calculus. I have a general doubt regarding the definition of least upper bound of sets. Let A be any set of real numbers and A is not a null set. Let S be the least upper bound of A. Then by definition "For every x belongs to A, x is lesser than or equal to S" Let M be an...
  41. P

    Supremum = least upper bound, anything > supremum?

    The supremum is defined as the "LEAST" upper bound. The word "least" makes me think, there is a "MOST" upper bound, or at least something bigger than a "least" upper bound. For a set of numbers, is there anything larger than a supremum? Supremum is analogous to a maximum, but I don't...
  42. P

    Am I understanding "supremum" correctly

    If let's say I have an expression: ##x\leq y## Since the supremum is defined as the "least upper bound," does this make sup(x) for this case ##x=y## or is it ##x = \infty##?
  43. Z

    Real Analysis - Infimum and Supremum Proof

    Hi Guys, I am self teaching myself analysis after a long period off. I have done the following proof but was hoping more experienced / adept mathematicians could look over it and see if it made sense. Homework Statement Question: Suppose D is a non empty set and that f: D → ℝ and g: D →ℝ. If...
  44. M

    Showing a sequence converges to its supremum

    Homework Statement : [/B]Let a = sup S. Show that there is a sequence x1, x2, ... ∈ S such that xn converges to a.Homework Equations : [/B]I know the definition of a supremum and convergence but how do I utilize these together?The Attempt at a Solution :[/B] Given a = sup S. We know that a =...
  45. O

    MHB Proof of Supremum of $M$ Mapping into Itself

    Let $f$ be a mapping of a metric space $M$ into itself. For $A\subset M$ let $\And(A)=sup\left\{d(a,b);a,b\in A\right\}$ and for each $x\in M$, let $O(x,n)=\left\{x,Tx,...{T}^{n}x\right\}$ $n=1,2,3...$ $O(x,\infty)=\left\{x,Tx,...\right\}$ Please prove that...
  46. O

    MHB Max A $\le$ Sup B: Is it True?

    for A$\subset$ B max A $\le$ sup B ? is it true ?
  47. S

    Infimum and supremum of empty set

    Hello, I can't wrap my mind around this: inf∅= ∞ sup∅= - ∞ Thank you in advance.
  48. R

    Rudin's Principles Theorem 1.11 (supremum, infimum)

    Mod note: Edited by removing [ sup ] tags. To the OP: Please don't fiddle with font tags, especially the SUP tag, which renders what you write in very small text (superscript). Hello everyone I have just started studying mathematics at university this summer and I have decided to supplement my...
  49. C

    Proof of Supremum of a Set

    Homework Statement 8. Let ##A## be a non-empty subset of ##R## which is bounded above. Define ##B = \{x ∈ R : x − 1 ∈ A\}##, ##C = \{x ∈ R : (x + 1)/2 ∈ A\}.## Prove that sup B = 1 + sup A, sup C = 2 sup A − 1. The attempt at a solution Note that ##sup A## exists. Let ##x ∈ B##; then ##x − 1...
  50. F

    MHB Supremum of $l_2$ Series: Is $\frac{1}{4^{n+1}}$ Correct?

    $Sup(\sum_{k=n+1}^{\infty}\frac{|x_{k}|^{2}}{4^{k}})$ where $x=(x_{1},x_{2},...)$ is in $l_{2}$ and the supremum is taken over all $x$ such that $||x||$=1. I think it is equal to $\frac{1}{4^{n+1}}$ Is this correct?
Back
Top